mshinners
Thanks Received: 135
Atticus Finch
Atticus Finch
 
Posts: 367
Joined: March 17th, 2014
Location: New York City
 
 
 

Re: Q21 - A new device uses the global

by mshinners Fri Dec 31, 1999 8:00 pm

Question Type:
Strengthen, though we're trying to strengthen a prediction instead of an argument.

Stimulus Breakdown:
Prediction: Ranchers will purchase this device at the current price
Facts: Outfitting all cows with the device is more expensive than alternatives

Answer Anticipation:
Interesting Strengthen question that reads more like an Explain a Result question. We'll need to either find a reason that ranchers will adopt this product despite the expense, or find a reason that the expense won't be as large as expected.

Correct answer:
(B)

Answer choice analysis:
(A) Out of scope. The prediction is about purchasing the item at the current price.

(B) Bingo. If this is true, the device needs to be installed on only some of the cattle. While outfitting each cow would be too expensive, the ranchers can get away with the lower cost of outfitting only some cows.

(C) Out of scope. The stimulus doesn't state that stress is a relevant consideration. Additionally, it's never stated what stress is caused by the other methods. Finally, the argument and prediction are about price.

(D) Out of scope. This answer, again, doesn't get at the cost effectiveness of it. If it's as effective as fences but more expensive, why wouldn't you just use fences?

(E) Out of scope. The prediction is about purchasing the item at the current price, not at a bulk-discounted price.

Takeaway/Pattern:
Be a bit flexible on question stems that are a little outside the norm! Even though this is a Strengthen prompt, treating it as an Explain question helps out.

#officialexplanation
 
yunjh2725
Thanks Received: 0
Vinny Gambini
Vinny Gambini
 
Posts: 5
Joined: September 25th, 2014
 
 
 

Q21 - A new device uses the global

by yunjh2725 Mon Nov 30, 2015 3:18 pm

Hi,

I chose E for the answer. I chose this because it said "Significant discounts."
I realize that it was given that "outfitting all of the cattle in a herd with this device is far more expensive..."
But, if there is SIGNIFICANT discount, wouldn't that explain why ranchers would consider buying?
I realize that B is a stronger answer since the rancher would only have to buy one... but how can one choose B over E?
Is B just a stronger answer than E in relations to the evidence given by the question (aka outfitting all would be too expensive regardless?)
Or, is E just a bad answer that I am seeing? the word Significant caught me and I didn't really see that "outfitting all cattle is expensive (regardless of any discounts)."

Thanks!
 
mcb220
Thanks Received: 0
Vinny Gambini
Vinny Gambini
 
Posts: 2
Joined: November 30th, 2015
 
 
 

Re: Q21 - A new device uses the global

by mcb220 Mon Nov 30, 2015 6:46 pm

Had trouble with the question as well. After going over it some more, I've considered two possibilities as to why E is wrong. Of course I may be wrong.

1) Although E claims that the purchasers would be offered 'Significant' discounts, it is possible that the overall price to equip every cow would still be for more expensive than simply installing a fence, even if a significant discount is provided.
For Example
20 Devices Prior to Discount: $1,000
20 Devices After Applying Discount: $700
Price to Install fence: $400

2) This may be a leap, but notice how the stimulus states that "Ranchers will purchase the device at its current price." So maybe the discount would not even apply. Again, this seems like huge leap.

I picked (E) for this question, and the most frustrating part is that when I read (B), I though to myself, "the LSAT would make this the right answer "
Any clarification for a geek would be greatly appreciated.
 
lampars2
Thanks Received: 0
Vinny Gambini
Vinny Gambini
 
Posts: 2
Joined: December 01st, 2015
 
 
 

Re: Q21 - A new device uses the global

by lampars2 Tue Dec 01, 2015 3:57 pm

Hi all,

I was going over this question in review and wanted to see what others thought of this Q, I choose answer choice B and believe I have the logic behind it so I figured I'd post my first explanation.

This is a strengthening question, therefore you must identify the conclusion and the premise(s).

Here we have the conclusion as, "The device maker nevertheless predicts that ranchers will purchase the device at its current price".

We have a premise that states "Outfitting all of the cattle in a herd with this device is far more expensive than other means of keeping cattle in their pastures, such as fences".

Upon reading this stimulus and before you even hit the question stem you should be thinking to yourself, if this device is far more expensive than fences and does the same thing why on earth would someone predict that the rancher would purchase such device?

After hitting the question stem and realizing this is a strengthening question, we must find an answer choice that would make us believe a rancher would choose this device over a cheaper solution such as a fence.

Answer choice B does exactly this. It states that cattle in a herd follow the same few leaders. Meaning that you could just place this device on the leader cattle and not on the majority of the cattle. So here we could infer that this premise would lower the overall cost of performing this task. Maybe even at a lower or equal cost to the cattle fence as mentioned in the stimulus.

All the other answer choices do not get at what we are trying to achieve. Hope this helps.
User avatar
 
maryadkins
Thanks Received: 640
Atticus Finch
Atticus Finch
 
Posts: 1261
Joined: March 23rd, 2011
 
 
 

Re: Q21 - A new device uses the global

by maryadkins Mon Dec 07, 2015 11:54 am

GREAT DISCUSSION HERE, GUYS! WOOHOO!

Yes, (E) negates the part where the prediction is that the device will be bought AT ITS CURRENT PRICE.

(B) is correct for the reason lampars2 gave.

(A) also negates the premise.

(C) - stress? So? Why are they going to buy it instead of a fence?

(D) AS effective? Okay, but it's still more $$, so why will it be bought?
 
jeanh93
Thanks Received: 0
Vinny Gambini
Vinny Gambini
 
Posts: 3
Joined: May 26th, 2016
 
 
 

Re: Q21 - A new device uses the global

by jeanh93 Thu May 26, 2016 10:36 am

I feel like most people narrowed down to two answer choices, B and E. I chose E, but as I look back and compare the two answer choices, I realize that E is more of an assumption compared to B, since for E we have to assume that the device maker will indeed offer significant discounts when purchasing the devices in bulk, whereas the statement in B is more of a fact that is inevitable.

E is also not true precisely because of the stimulus that says, "outfitting all of the cattle in a herd with this device is far more expensive than other means." This stimulus essentially tells us that even if the ranchers could buy devices much cheaper than the market price, the purchase would still be far more expensive than utilizing fences, hence it doesn't strengthen the prediction in any way.
User avatar
 
snoopy
Thanks Received: 19
Elle Woods
Elle Woods
 
Posts: 70
Joined: October 28th, 2017
 
 
 

Re: Q21 - A new device uses the global

by snoopy Mon Jun 11, 2018 8:36 am

I also narrowed to B and E but chose E. I eliminated B because I wasn't sure if placing the devices in some cattle would ensure all or most cattle in the herd would follow the leader cows. I pictured some cows getting lost along the way. What's wrong with this thinking?
User avatar
 
ohthatpatrick
Thanks Received: 3805
Atticus Finch
Atticus Finch
 
Posts: 4661
Joined: April 01st, 2011
 
This post thanked 1 time.
 
 

Re: Q21 - A new device uses the global

by ohthatpatrick Mon Jun 11, 2018 4:06 pm

Just to reiterate the reason (E) is ineligible:
an answer only works if it helps us to believe that the ranchers will purchase the device AT ITS CURRENT PRICE.

Buying the device AT A SIGNIFICANT DISCOUNT has nothing to do with the claim we're trying to support.

In order for (E) to be worth anything to this story, we'd be telling ourselves that the ranchers will be okay buying devices for all of their cattle because they'll be buying the device as a significantly discounted price.

But to get a correct answer, we need a story where the ranchers buy the device at the current price. So (E) is totally irrelevant to our task.

Your reservations with (B) are that it isn't perfect; it isn't bulletproof.

You want to be using a much looser standard of judgment for
STRENGTHEN
WEAKEN
EXPLAIN/RESOLVE

These questions are all structured as,
"Which of the following, if true, does the most ..."

An answer doesn't have to score a touchdown to be correct. It can be a 20 yd gain or even a puny 1 yd gain. If it does more than any other answer, then it DOES THE MOST.

(B) is the only answer that gives us a coherent story:
The ranchers will by the device at the current price because they only need to buy a few of them; the cattle in a herd follow the lead of those same few members.

(Even though our answer doesn't need to be bulletproof, it's worth noting that the wording of B is not hedged in any way. It says 'cattle in a herd' follow the lead. That's a categorical statement. It conveys the idea that ALL of them follow the lead)